4
$\begingroup$

I'm wondering if analytic number theorists can prove results which have the following flavor:

So let $N$ be a large positive integer.

Q: Can you always find a prime number $p$ in the interval $(N, 3N/2)$ for which there exists an odd prime $q$ which divides $p-N$ with multiplicity exactly one?

If such a result can be found in the literature I would like to have a reference. I have just not the single idea about where to start in order to prove such a result.

I kind of remember vaguely that every large enough even integer $N$ can be written as $p_1+p_2p_3$ where the $p_i$'s are prime numbers which is not that far from what I'm asking for.

$\endgroup$
2
  • 1
    $\begingroup$ You're asking for primes p in arithmetic progressions (say with modulus the square of q)? This is much easier than approximations to Goldbach, unless the intervals are very short. $\endgroup$ Mar 20, 2011 at 22:57
  • 1
    $\begingroup$ Following on Charles' comment, let $q$ be the smallest odd prime not dividing $N$, then there should be lots of primes $p$, $N\lt p\le 3N/2$, $N\equiv p\pmod q$, $N\not\equiv p\pmod{q^2}$. Time to rummage through the literature on primes in arithmetic progressions. $\endgroup$ Mar 20, 2011 at 23:31

2 Answers 2

6
$\begingroup$

The number of primes in $[N,3N/2]$ grows as $\frac{N}{\log N}$, while the number of powerful numbers in $[1,N/2]$ grows as $\sqrt{N}$, so pretty quickly you will find primes $p\in [N,3N/2]$ so that $p-N$ is not powerful, i.e. has a prime divisor which has multiplicity 1.

$\endgroup$
4
  • $\begingroup$ Also note that the asymptotic results needed come with effective constants so this will hold for all $N>c$ for some $c$ we can compute. $\endgroup$ Mar 21, 2011 at 0:36
  • 1
    $\begingroup$ In particular, combining Golomb 1970 with Rosser 1941 allows an explicit (and small) bound on allowable N. $\endgroup$
    – Charles
    Mar 21, 2011 at 0:39
  • $\begingroup$ @Gjergji Zaimi: Sorry, didn't see your comment until I added mine. Yes, same idea. $\endgroup$
    – Charles
    Mar 21, 2011 at 0:40
  • $\begingroup$ Thanks a lot Gjergji, I might need some effective constant at some point. $\endgroup$ Mar 21, 2011 at 3:32
1
$\begingroup$

I cannot think of an exact reference but the result you are looking for can be obtained as follows:

The number of primes $p\in(N,2N]$ such that $p-N$ is divisible by the square of a prime $q>\log N$ is $\ll N/\log^2N$ (this follows by any upper bound sieve). Also, the number of primes $p\in(N,2N]$ such that $p-N$ is composed only of prime numbers $\le\log N$ is at most the number of integers $m\le N$ which are $\log N$-smooth (i.e. have only only prime factors $\le\log N$). The number of such integers is at most $N^{1-1/2\log\log N}\ll N/\log^2N$ (see for example Theorem 1, Chapter III.5, in Tenenbaum's book "Introduction to analytic and probabilistic number theory"). So

$$|\{N < p\le2N:\exists q~{\rm prime}~{\rm with}~q>\log N~{\rm and}~q\|p-N\}|=\frac N{\log N}+O\Bigl(\frac N{\log^2N}\Bigr).$$

$\endgroup$

Your Answer

By clicking “Post Your Answer”, you agree to our terms of service and acknowledge you have read our privacy policy.

Not the answer you're looking for? Browse other questions tagged or ask your own question.